Aditividad de la función de entropía

Estoy leyendo el libro Termodinámica de Herbert Callen, que propone un tratamiento postulatorio del tema.

El postulado número 3 establece las propiedades de la función de "entropía", una de las cuales es la propiedad aditiva: la entropía de un sistema compuesto por múltiples subsistemas separados por restricciones internas es la suma de la entropía de los subsistemas individuales.

Considere dos sistemas con energías internas tu 1 , tu 2 , volúmenes V 1 , V 2 y número de moléculas norte 1 , norte 2 . ¿No significa la afirmación anterior

S ( tu 1 + tu 2 , V 1 + V 2 , norte 1 + norte 2 ) = S ( tu 1 , V 1 , norte 1 ) + S ( tu 2 , V 2 , norte 2 )

? En cuyo caso la función de entropía es lineal. Pero la función de entropía para un gas ideal no es lineal en el sentido anterior.

Para dos sistemas idénticos, significaría que la función de entropía debe ser una función homogénea de primer orden, lo cual tiene sentido. Pero esto no implica linealidad.

Entonces, me gustaría saber el lapso en mi entendimiento. ¡Gracias!

Editar: la ecuación anterior no califica para que la función de entropía sea lineal. Creo que la aditividad de la entropía se puede representar matemáticamente mediante esta ecuación.

Respuestas (2)

Creo que más bien significa:

S t o t ( tu 1 , tu 2 , V 1 , V 2 , norte 1 , norte 2 ) = S 1 ( tu 1 , V 1 , norte 1 ) + S 2 ( tu 2 , V 2 , norte 2 )
Piense, por ejemplo, en dos contenedores aislados. Si la afirmación, tal como la escribiste, fuera cierta, la mezcla de dos gases ideales sería reversible.

Muchas gracias. ¡Ahora parece que no puedo recordar por qué estaba pensando de esa manera!

Una respuesta tardía, que amplía la respuesta anterior de denklo.

Si los dos subsistemas están hechos del mismo material, la aditividad de los volúmenes, energías y número de partículas permite escribir

S t o t ( tu 1 + tu 2 , V 1 + V 2 , norte 1 + norte 2 ; tu 1 , V 1 , norte 1 ) = S ( tu 1 , V 1 , norte 1 ) + S ( tu 2 , V 2 , norte 2 ) ,
donde la dependencia de 6 argumentos de S t o t aclara que i) la entropía del sistema compuesto no es la misma función, en general, que la entropía de cada subsistema; ii) que existe una dependencia de la entropía total de las restricciones de energía fija, volumen y número de partículas en cada subsistema, que puede codificarse en el valor de esas cantidades en uno solo de los dos subsistemas, ya que su suma es fija .

Escrito de esta forma, el efecto de eliminar todas las restricciones implica maximizar S t o t con respecto a las variables de restricción ( tu 1 , V 1 , norte 1 ). Las condiciones extremas resultantes expresan la condición de temperatura, presión y potenciales químicos iguales en los dos subsistemas.

Por lo tanto, es trivial ver que solo en el caso de equilibrio termodinámico

S t o t ( tu 1 + tu 2 , V 1 + V 2 , norte 1 + norte 2 ; tu 1 , V 1 , norte 1 ) = S ( tu 1 , V 1 , norte 1 ) + S ( tu 2 , V 2 , norte 2 ) = S ( tu 1 + tu 2 , V 1 + V 2 , norte 1 + norte 2 ) .
En efecto, basta recordar que la homogeneidad de grado uno de la entropía de los dos subsistemas en el equilibrio implica
S ( tu 1 , V 1 , norte 1 ) = tu 1 T + V 1 PAG T norte 1 m T S ( tu 2 , V 2 , norte 2 ) = tu 2 T + V 2 PAG T norte 2 m T
de donde se sigue la conclusión.